bosefermiのブログ

更新が面倒に感じてきた

1次 計算技能検定 問題5

問題は以下のリンクから確認できます。

※今回のシリーズは2021年9月13日に確認したもので、記載される問題が変更されている可能性があります。

www.su-gaku.net

 

【問題5】

0以上の整数値をとる確率変数Xが下の確率分布に従うとき、次の問いに答えなさい。ただし\begin{pmatrix} n\\r \end{pmatrix}は二項係数を表します。

P(X=k)=\displaystyle \begin{pmatrix} n\\r \end{pmatrix}\left(\frac{1}{10}\right)^2\left(\frac{9}{10}\right)^k・\frac{1}{10}

Xの期待値E(X)を求めなさい。

Xの分散V(X)を求めなさい。

 

【解答例】

P=\displaystyle \frac{1}{2}・\left(\frac{1}{10}\right)^3・\sum_{k}(k+2)(k+1)\left(\frac{9}{10}\right)^k

(多項式)×(等比)の形なので9/10倍でずらすことを考えていく。

E(X)=\displaystyle \sum_{k} kP= \frac{1}{2}・\left(\frac{1}{10}\right)^3・\sum_{k}(k+2)(k+1)k\left(\frac{9}{10}\right)^k

I=\displaystyle\sum_{k}(k+2)(k+1)k\left(\frac{9}{10}\right)^kとおく。

I=\displaystyle 0+3・2・1・\frac{9}{10}+4・3・2・\left(\frac{9}{10}\right)^2+・・・\\ \displaystyle \frac{9}{10}I=\hspace{53pt}0+3・2・1・\left(\frac{9}{10}\right)^2+・・・

引き算して

 \displaystyle \frac{1}{10}I=\sum_{k} \left\{\left(k+3\right)\left(k+2\right)\left(k+1\right)-\left(k+2\right)\left(k+1\right)k\right\}\left(\frac{9}{10}\right)^{k+1}\\=\displaystyle 3\sum_{k} (k+2)(k+1)\left(\frac{9}{10}\right)^{k+1}

よって\displaystyle \sum_{k} P=1に注意して

\displaystyle E(X)=\frac{1}{2}・\left(\frac{1}{10}\right)^3・I\\ \displaystyle =10・3・\frac{1}{2}・\left(\frac{1}{10}\right)^3・\sum_{k} (k+2)(k+1)\left(\frac{9}{10}\right)^{k+1}\\ \displaystyle =10・3・\frac{9}{10}=27

 

次に分散の計算を(二乗の平均)-(平均の二乗)を用いて行う。先の問題同様にして二乗の平均は計算する。

E(X^2)=\displaystyle \sum_{k} k^2P= \frac{1}{2}・\left(\frac{1}{10}\right)^3・\sum_{k}(k+2)(k+1)k^2\left(\frac{9}{10}\right)^k

J=\displaystyle\sum_{k}(k+2)(k+1)k^2\left(\frac{9}{10}\right)^kとおく。

引き算すると

 \displaystyle \frac{1}{10}J=\sum_{k} \left\{\left(k+3\right)\left(k+2\right)\left(k+1\right)^2-\left(k+2\right)\left(k+1\right)k^2\right\}\left(\frac{9}{10}\right)^{k+1}\\=\displaystyle \sum_{k} (k+2)(k+1)(4k+3)\left(\frac{9}{10}\right)^{k+1}

よって4k+3の展開のうち4kの項は前問と比較してE(X)の定数倍、3の項は確率和1の定数倍になることがわかるので

\displaystyle E(X^2)=\left(4\sum_{k} kP+3\sum_{k} P\right)・\frac{9}{10}・10=(4・27+3)・10・\frac{9}{10}=37・27

また、(E(X))^2=27・27であるから

V(X)=37・27-27・27=10・27=270

 

【コメント】

tex記法で書くので大変。